DSM-5 Study Guide Questions and Answers

Pataasin ang iyong marka sa homework at exams ngayon gamit ang Quizwiz!

Aphasia

Loss of impairment of the power to use or comprehend words usually resulting from brain damage.

Match each time criterion with the most appropriate diagnoses (each time criterion may be used one, more than once, or not at all): A. Symptoms persist less than 1 month. B. Symptoms persist more than 1 month. _ acute stress disorder. _ Delusional disorder. _ Posttraumatic stress disorder. _ Brief psychotic disorder.

A. Acute stress disorder- symptoms persist less than 1 month. B. Delusional disorder- symptoms persist more than 1 month. B. Posttraumatic stress disordder- symptoms persist more than 1 month. A. Brief psychotic disorder- symptoms persist less than 1 month.

Echolalia

The pathological, parrotlike, and apparently senseless repetition (echoing) of a word or phrase just spoken by another person.

A 26- year- old man seen in the emergency department acknowledges persistent substance use until about the last 2 days, when he was evicted from his apartment. Which cluster of symptoms is typical of stimulant withdrawal? A. Depression, fatigue, insomnia. B. Diarrhea, nausea, anxiety. C. Hypertension, tachycardia, seizures. D. Vivid dreaming, nightmares, confusion.

A. Depression, fatigue, insomnia.

Match each description with the most appropriate diagnosis (each disorder may be used once, more than once, or not at all): A. Bipolar I. B. Bipolar II. C. Cyclothymia. _ Meets criteria for mania. _ Meets criteria for hypomanic but not mania. _ does not meet criteria for hypomanic or mania.

A. Bipolar I- Meets criteria for mania. B. Bipolar II- Meets criteria for hypomanic but not mania. C. Cyclothymia- Does not meet criteria for hypomanic or mania.

Partners in a longer- term affectionate and intimate relationship have recently begun to "experiment" with wearing costumes with especially soft fabrics and engaging in role- playing during their sexual encounters. They are enjoying this practice and find that it enriches their sexual life together. Does this pattern meet criteria for fetishistic disorder? A. No, b/c neither partner is experiencing distress. B. No,, b/c soft fabrics do not represent a fetishistic object. C. Yes, b/c role- playing represents a fetish. D. Yes, b/c this "experimentation" is new.

A. No, b/c neither partner is experiencing distress.

Agnosia

Loss of ability to recognize objects persons, sounds, shapes, or smells that occurs in the absence of either impairment of the specific sense or significant memory loss.

Apraxia

Loss of impairment of the ability to execute complex coordinated movements without muscular or sensory impairment.

Which statement is correct regarding the domains DSM-5 evaluates in the Clinician- Rated Dimensions of Psychosis Symptoms Severity? A. The scale assesses only the three most important domains of hallucinations, delusions, and disorganized speech. B. The scale is based on symptoms experienced by the pt in the previous 7 days. C. The scale is most accurate if pts, rather than clinicians, provide the responses. D. The scale should only be completed if the clinician is certain of the symptom severity.

B. The scale is based on symptoms experienced by the pt in the previous 7 days.

Using the DSM-5 criteria to "diagnose" celebrities on the basis of media reports is not considered ethically acceptable in the profession for many reasons, but especially for which reasons? A. Celebrities do not like to be diagnosed. B. DSM-5 diagnostic criteria do not apply to celebrities. C. DSM-5 diagnostic criteria rely on information gathered through a comprehensive evaluation. D. Medical do not accurately portray celebrities.

C. DSM-5 diagnostic criteria rely on information gathered through a comprehensive evaluation.

A primary care physician refers a 60- year- old Cambodian man to a psychiatrist to rule out a psychotic disorder. The man was previously given a diagnosis of Schizophrenia b/c he "hears voices". He has been prescribed a large dose of an antipsychotic medication, which he talked daily, with very little symptom relief. He states that the medicine helps him sleep. The psychiatrist learns that the man is a survivor of the Cambodian genocide of the 1970s who survived torture under the Khmer Rouge regime, witnessed executions, and endured forced labor in Communist Cambodia. The man reports that subsequent to the extremes of traumatic experience he sustained, he has had chronic symptoms of anxiety with panic attacks. He also reports a concern of "thinking too much". When asked to clarify what he means, he describes ruminating, worried thoughts about his past traumas. He reports severe insomnia and nightmares about Cambodia at night. He describes hearing voices other people do not hear. Further questioning reveals that the voices he hears are of Khmer Rouge officers, and he "hears" them when he is remembering his experiences of violence. When the psychiatrist asks him to clarify this, he says the "voices" are more like memories than hallucinations. What is the likely diagnosis? A. Bipolar I disorder, current episode manic. B. Panic disorder. C. Posttraumatic stress disorder. D. Schizophrenia.

C. Posttraumatic stress disorder. This disorder is defined as exposure to actual or threatened death, serious injury, or sexual violence in one or more of the following ways: directly experiencing a traumatic event, witnessing the event that occurred to others, learning that the traumatic event occurred to a close family member or close friend, experiencing repeated or extreme exposure to aversive details of the traumatic events. The symptoms have occurred for over a 1- month period. The individual has increased arousal and avoids internal and external stimuli that are reminiscent of the trauma. There are specific criteria for children 6 years and younger.

Which statement is correct regarding how DSM-5 diagnoses and documents personality disorders? A. DSM-5 has eliminated personality disorder diagnoses. B. DSM-5 has introduced new personality disorder subyptes. C. DSM-5 has maintained the use of the Axis II system for documenting personality disorders. D. DSM-5 has preserved the same diagnostic criteria as in DSM-IV-TR for personality disorders.

D. DSM-5 has preserved the same diagnostic criteria as in DSM-IV-TR for personality disorders.

DSM-5 assigned to new diagnostic classes some diagnoses that DSM-IV included in disorders usually first diagnosed in infancy, childhood, or adolescnce. Which of the following statements is correct regarding these changes? A. Selective mutism was added to anxiety disorders. B. Selective mutism was added to depressive disorders. C. Separation anxiety disorder was added to dissociative disorders. D. Separation anxiety disorder was added to schizophrenia spectrum and other psychotic disorders.

A. Selective mutism was added to anxiety disorders.

To meet criteria for premenstrual dysphoric disorder, how long must symptoms be present? A. 1 week. B. 1 month. C. 6 months. D. 1 year.

D. 1 year.

For a diagnosis of bipolar I disorder, an individual must have experienced in the past which of the following events, even if the event was not formally documented or diagnosed Andy did not lead to an episode of clinical care? A. Depressive episode. B. Head injury. C. Manic episode. D. Psychological trauma.

C. Manic episode.

A young girl is in second grade. Last year her parental grandfather died from cancer. Since then her parents have become increasingly concerned about her. Last week a babysitter said that the girl sat staring at the front door almost the entire night, waiting for her parents to return from their date night. Their daughter keeps asking to say home from schoo. She pleads for her mother to stay home with her, stating, "I don no wan you to drive to work. What if you get in an accident? Just stay home with me." Before she goes to bed at night, she complains of headaches, which only seem to subside if she sleeps in her parent's room. They have stopped allowing her to sleep in their room, but some mornings they find her sleeping in the hallway right outside their bedroom door. She refuses to attend sleepovers with friends b/c she always feels "sick" while she is there. What is the likely diagnosis? A. Major depressive disorder. B. Panic disorder and agoraphobia. C. Separation anxiety disorder. D. Social anxiety disorder.

C. Separation anxiety disorder. This disorder is distinguished by developmentally inappropriate excessive anxiety around separation from significant others (parents or spouse). Features include clinically significant (e.g severe distress or impairment of function) symptoms of anxiety; unrealistic worries about the safety of loved ones; reluctance to fall asleep without being near the primary attachment figure; excessive distress, such as tantrums, when separation is imminent; nightmares with separation- related themes; and homesickness, such as a desire to return home or make contact with the significant others. In addition, physical and somatic symptoms, such as dizziness, lightheadedness, headaches, names, stomachache, cramps, vomiting, muscle aches, or palpitations, may be present and problematic. In children and adolescents the symptoms must be present for at least 4 weeks and in adults 6 months. Symptoms cause significant impairment and distress.

A 19- year- old college student is brought to the psychiatric emergency department by his friends b/c he had not slept for 4 days, was speaking loudly, felt "exuberant" and "on top of the world", and b/c he had attempted to lift a care with his "bare hands". He was an avid weight lifter and recently had stated taking steroids (purchased "on the streets") to help him "bulk up". What is the most appropriate diagnosis? A. Bipolar I. B. Bipolar II. C. Substance/ medication- induced Bipolar and related disorder. D. Substance/ medication- induced psychotic disorder.

C. Substance/ medication- induced bipolar and related disorder.

Which statement is correct regarding the depressive disorders diagnostic class of DSM-5 in comparison with earlier versions of DSM? A. Brief depressive disorder has been added as a new diagnosis. B. Disruptive mood dysregulation disorder has been eliminated as a diagnosis. C. Persistent depressive disorder has been eliminated as a diagnosis. D. Premenstrual dysphoric disorder has been added as a new diagnosis.

D. Premenstraul dysphoric disorder has been added as a new diagnosis.

Match each description with the paraphilic disorder for which it is most highly characteristic (each disorder may be used once, more than once, or not at all): A. Exhibitionistic disorder. B. Fetishistic disorder. C. Pedophilic disorder. D. Sexual sadism disorder. E. Voyeuristic disorder. _ Involves exposing one's genitalia to unsuspecting people. _ Involves intense sexual arousal from observing an unsuspecting person disrobing or being naked. _ Involves intense sexual arousal from prepubescent children. _ Involves intense sexual arousal from the significant physical or psychological suffering of another person.

A. Exhibitionisitic disorder- Involves exposing one's genitals to unsuspecting people. E. Voyeuristic disorder- Involves intense sexual arousal from observing an unsuspecting person disrobing or being naked. C. Pedophilic disorder- Involves intense sexual arousal from prepubescent children. D. Sexual sadism disorder- Involves intense sexual arousal from the significant physical or psychological suffering of another person.

A. 40- year- old woman sees a psychiatrist for the first time. She complains of having had low energy and fatigue for the past year and endorses additional symptoms of depressed mood, poor sleep, and decreased concentration. She exhibits significant psychomotor retardation on exam. She denies any suicidal ideation or hallucinations. She takes medication for hypothyroidism but has not seen a primary care doctor in over 10 years. Her thyroid- stimulating hormone level is 7.6, and her urine toxicology screen is negative. What is the likely diagnosis? A Bipolar II disorder. B Depressive disorder due to another medical condition. C. Insomnia disorder. D. Major depressive disorder.

B. Depressive disorder due to another medical condition.

A husband brings his 35- year- old wife to a psychiatrist b/c he things she has been buying a lot of jewelry over the past few years. In private, she admits that she stole all her jewelry from department stores. She describes feeling excited by Te prospect of stealing and then gratified after each theft. She denies hallucinations or mood disturbances or any other motives for acquiring the jewelry. She denies any distress associated with the desire to steal or the act of theft. On exam, her though process is linear with fluent, normal speech, and no evidence of psychomotor agitation. She denies any problems with sleep. What is the likely diagnosis? A. Bipolar I disorder. B. Delusional disorder. C. Kleptomania. D. Obsessive- compulsive disorder.

C. Kleptomania. This disorder is a repeated failure to resist the impulse to steal items the individual does not have reason to steal. The theft is not a way of expressing anger or getting vengeance and is not related to a delusion or a hallucination. The behavior is associated with increased tension just before the theft and a sense of pleasure or relief in the satisfaction o ft he impulse. Most of the individuals with this disorder are woman.

A wife brings her husband, a 55- year- old man who has chronic conditions of diabetes, obesity, and hypercholesterolemia to his primary care doctor. She is upset that her husband has no energy to go on outings wit her and that he is always tired; she ways that even though he would like to do things with the family, he consistently declines to do so. She gave him her own prescription for sleeping medication ("I know I should not do that! I was just hoping it would help us!"), but all that she noted afterwards is the he snored "more than usual". She is worried about the well- being of their marriage, given her husband's changes. The man notes that he is interested in life, their marriage, going out to dinner, and so on, but the does not know whey he feels so exhausted. What is the likely diagnosis? A. Adjustment disorder. B. Anorexia. C. Major depressive disorder. D. Obstructive sleep apnea hypopnea.

D. Obstructive sleep apnea hypopnea. Is the most common type of breathing related sleep disorder. It is characterized by a total absence of breathing (apnea) or decrease in airflow (hypopnea). When the brain registers impaired breathing it briefly arouses the individual from sleep. Snoring is a central features of this breathing related sleep disorder.

A 28- year- old woman seeks psychiatric consultation. She explains that 10 years ago, she had a panic attack while presenting her project at the high school science fair. Since then she has developed an intense fear of being embarrassed or humiliated in a public venue. She is constantly worried that others are scrutinizing her. She is very self- conscious and afraid that she will have heart palpitations, trembling, or stuttering or that her mind will go blank. She refuses to go on dates and is not being considered for a new job b/c she declined the interview. She is tired of living such an isolated life. What is the likely diagnosis? A. Avoidant personality disorder. B. Obsessive- compulsive disorder. C. Panic disorder. D. Social anxiety disorder (social phobia).

D. Social anxiety disorder (social phobia). This disorder is characterized by fear of embarrassing oneself in social situations or feeling foolish. Avoidance behavior is often evident and the condition is not due to a medical situation or substance. The disorder typically lasts for more than 6 months.

A diagnosis of conduct disoder, childhood- onset type, has been associated with preceding which of the following disorders? A. Antisocial personality disorder. B. Attention- deficit/ hyperactivity disorder. C. Narcissistic personality disorder. D. Oppositional defiant disorder.

A. Antisocial personality disorder.

Police arrest a 26- year- old man who got into a fight in a bar after her was discovered in the act of stealing the wallet of another customer. He defended his behavior to the police, stating that the other man had provoked him earlier in the evening by calling him a profane name. Review of his criminal record reveals charges, including some as a minor, for credit card fraud and seeking cocaine. He lives alone and is currently unemployed. He has no psychiatric hx. He does not report any recent hallucinations or changes in his mood or sleep. What is the likely diagnosis? A. Antisocial personality disorder. B. Conduct disorder. C. Moderate cocaine use disorder. D. Narcissistic personality disorder.

A. Antisocial personality disorder. This disorder is characterized by a pattern of disregard for or violation of others' rights evident since at least age 15 and the individual is at least 18 years of age. If the individual's behavior is consistent with this diagnosis, but he or she is younger than 18, the diagnosis of conduct disorder should be used. This disorder is more common in males.

Women account for more than 90% of the diagnoses for which of the following disorders? A. Bulimia nervosa. B. Major depressive disorder. C. Obsessive- compulsive disorder. D. Schizoprehnia.

A. Bulimia nervosa.

The neurology department refers a 39- year- old woman to psychiatry b/c she has been experiencing tonic- clinic movements of extremities without loss of consciousness for the past 2 months. An extensive neurological workup has been preformed and no abnormalities are evident. The pt is cooperative throughout the session but shows restricted range of affect. Her husband says that she has been under treatment for infertility for many years and miscarried during a much desired pregnancy about 3 months ago. She does not show significant changes in her affect in reaction to this information from he husband. What is the likely diagnosis? A. Conversion disorder (functional neurological symptom disorder). B. Depressive disorder due to another medical condition. C. Factitious disorder. D. Somatic symptom disoder.

A. Conversion disorder (functional neurological symptom disorder). This disorder is characterized by loss of functioning in voluntary motor and sensory functions deemed not to be fully the result of a medical condition, a substance, or a phenomenon sanctioned by a culture. The loss of functioning is initiated or exacerbated by a stressful event and is not caused exclusively by pain or sexual dysfunction. The disorder is more common in women than in men. The longer the duration of the individual's symptoms and the more regressed the individual is, the more difficult is the treatment.

Match each description with the sexual dysfunction diagnosis for which it is most highly characteristic (each disorder may be used one, more than once, or not at all): A. Erectile disorder. B. Female orgasmic disorder. C. Genito- pelvic pain/ penetration disorder D. Premature (early) ejaculation _ involves inability to attain and maintain arousal. _ Involves inability to have vaginal intercourse or penetration. _ involves inorgasmia or delayed orgasm. _ Involves orgasm before the person desires.

A. Erectile disorder- Involves inability to attain and maintain arousal. C. Genito- pelvic pain/ penetration disorder- Involves inability to have vaginal intercourse or penetration. B. Female orgasmic disorder- Involves inorgasmia or delayed orgasm. D. Premature (early) ejaculation- Involves orgasm before the person desires.

A 53- year- old woman presents for a psychiatric evaluation of her long- standing depression. She has has several depressive episodes and was once hospitalized after a serious suicide attempt. She describes sexual abuse by her father and neglect by her mother during her teenage years. During the interview, while talking about her childhood, she suddenly switches to talking in a childlike, high- pitched voice, while pulling her legs under her in a chair and starting to rock backward and forward. While rocking, she says, "Don't hurt me, don't hurt me". Later during the interview, when she is talking and behaving in her original manner, she endorses not being able to recount her activities during several hours per day on a number of occasions in the past year. What is the likely diagnosis? A. Major depressive disorder with dissociative identity disorder. B. Major depressive disorder and Schizoaffective disorder. C. Major depressive disorder and schizophrenia. D. Major depressive disorder and somatic symptom disorder.

A. Major depressive disorder with dissociative identity disorder.

A 33- year- old man presents to a psychiatrist with concern about his excessive hand washing. He reports that he has a ritualistic washing routine that takes at least 20 minutes, which he performs multiple times per day for fear of contamination with infectious disease. When further interviewed, he states that he has recurrent and persistent thoughts about hand washing and risk of contamination, which he tries unsuccessfully to suppress. He reports an overwhelming sense of anxiety if he does not wash his hands or if he does not complete the "ritual". He says the behavior is "out of control". He recognizes these thoughts as functions of his own mind. He has no other symptoms. What is the likely diagnosis? A. Obsessive- compulsive disorder. B. Obsessive- compulsive personality disorder. C. Schizophrenia D. Specific phobia.

A. Obsessive- compulsive disorder. This disorder is characterized by anxiety- producing obsessions (persistent thoughts, urges, or images) or compulsions (behavior's the individual feels compelled to do repeatedly according to rigid rules). The obsessions or compulsions are time- consuming or cause clinically significant distress or impairment in social, occupational or other important areas of functioning.

Match each mental disorder with the age at onset noted in the diagnostic criteria: A. Age at onset under 5 years. B. Age at onset under 18 years. C. Age at onset not specified. _ Tourette's disorder. _ Separation anxiety disorder. _ Global developmental delay.

B. Age at onset under 18 years- Tourette's disorder. C. Age at onset not specified- Separation anxiety disorder A. Age at onset under 5 years- Global developmental delay.

A 20- year- old college freshman is brought to the attention of the dormitory resident assistant b/c she is not attending meals in the dining hall. The student says she feels anxious about being in the crowded kitchen and dining hall. Similarly, she has ordered all her schoolbooks online to avoid being in the bookstore b/c "it is too overwhelming". She also has not attended some of her crowded lectures. The student is able to have a good conversation in the hall of the dormitory with the resident assistant. The student denies fearing scrutiny by others. She explains that she avoids some places b/c she found that in malls or other crowded ares, she feels very worried, faints, sweaty, and dizzy. She is afraid of losing control and in being in situations where she cannot escape. What is the likely diagnosis? A. Acute stress disorder. B. Agoraphobia. C. Posttraumatic stress disorder. D. Social anxiety disorder.

B. Agoraphobia. Is characterized by fear and anxiety in two or more of the following five situations: using public transportation, being in open spaces, being in enclosed places, standing in line or being in a crowd, being outside of the home alone. The individual fear or avoids these situations b/c of thoughts that escape might be difficult or help might not be available

A child is struggling in second grade. Her teacher called her parents for a conference due to concerns that she has become forgetful in class, is often seen to be daydreaming, and has serious difficulty following multistep directions. Her mother noted that "in second grade there is some real homework and she cannot sit down and focus on doing it. Half the time she even forgets it at home in the morning." Her teacher recalls careless errors on smile mathematics assignments. The child's father reminds them, "She has been this way for years. You cannot get her to focus, even if it is on making her bed or brushing her teeth." What is the likely diagnosis? A. Attention- deficit/ hyperactivity disorder, predominantly hyperactivity-impulsivity presentation. B. Attention- deficit/ hyperactivity disorder, predominantly inattentive presentation. C. Conduct disorder. D. Major depressive disorder.

B. Attention- deficit/ hyperactivity disorder, predominantly inattentive presentation. This disorder has a clear pattern of in attention and/ or hyperactivity not accounted for by an individual's developmental stage. ADHD is more common in males. Some symptoms are evident before age 12 and impairment must be significant and evident in at least two different settings. In preschool children, hyperactivity is the main manifestation. In school age children, in attention is more prominent. In adolescents, hyperactivity is less common and may be more related to figetiness, inner restlessness or impatience. In adulthood, impulsivity may remain problematic even when hyperactivity has diminished. In children, there must be at least 6 symptoms in either or both in attention and hyperactivity- impulsivity. In adolescents and adults (over age 17) the individual must present with five symptoms.

A 16- year- old boy presents to his high school counselor after running out of his programming class b/c he was asked to explain a piece of code. He has trouble answering and had felt this sort of difficulty in the past, but today it was worse. He is not able to explain clearly to the counselor what happened b/c he is still having the same speaking problems that he has in class. He speaks in broken words and keeps getting stuck on sounds like "aye" and "ah". He seems to spit out words. He writes down on a piece of paper that he has been thinking about dropping out of school b/c of his difficulty speaking; his older brother dropped out of school for similar problems. The counselor later observes him working in the computer lab and notices no abnormal behavior. What is the likely diagnosis? A Autism spectrum disorder. B. Childhood- onset fluency disorder (stuttering). C. Social anxiety disorder (social phobia). D. Tourette's disorder.

B. Childhood- onset fluency disorder (stuttering). Is diagnosed when a child fails to use age and dialect appropriate speech sounds. The average age of onset is by age 6, but the range can be b/w 2-7 years of age. The disorder causes significant anxiety related to speaking or limitations in effective communication. Stress and anxiety can exacerbate the disroder. The disorder interferes with the individual's achievement in academics, occupation, and interpersonal communication. The individual may avoid situations due to fear that stem from humiliation and embarrassment.

For which of the following DSM- 5 diagnoses is there a far greater prevalence in men than women? A. Depersonalization/ derealization disorder. B. Exhibitionistic disorder. C. Major depressive disorder. D. Narcolepsy.

B. Exhibitionistic disorder.

Match each description with the appropriate phrase (each item may be used once, more than once, or not at all): A. False negative diagnosis. B. False positive diagnosis. C. True net active diagnosis. D. True positive diagnosis. _ A woman receives the diagnosis of bipolar disorder, manic phase, when her clinical presentation was actually due to amphetamine intoxication. _ An elderly man receives no psychiatric diagnosis 2 months after the death of his wife; he sometimes is tearful and sad and once each week has difficulty sleeping when he thinks of her.

B. False positive diagnosis - A woman receives the diagnosis of bipolar disorder, manic phase, when her clinical presentation was actually due to amphetamine intoxication. C. True negative diagnosis- An elderly man receives no psychiatric diagnosis 2 months after the death of his wife; he sometimes is tearful and sad and one each week has difficulty sleeping when he thinks of her.

A 37- year- old single mother divorced her husband about 8 months ago and has custody of their two daughters. She went back to work full time "to make ends meet". About 2 moths ago, while at work, she began to feel extremely anxious. Within a few minutes, she had heart palpitations and difficulty breathing and almost fainted. Her coworkers rushed her to the hospital. A thorough medical workup was performed and no underlying medical cause was identified. Within 1 week, she had another attack at the local grocery store and felt embarrassed b/c she has to lie down on the floor for about 10 minutes while she was surrounded by the store's concerned customers until the symptoms subsided. She has had two other "freak outs" in different situations. She has constant concern that the problem will "happen again". She has been skipping meetings at work to avoid embarrassment in front of her coworkers in case she "goes crazy" again. What is the likely diagnosis? A. Generalized anxiety disorder. B. Panic disorder. C. Social anxiety disorder (social phobia). D. Specific phobia.

B. Panic disorder. This disorder is characterized by a pattern of recurrent panic attacks accompanied by persistent worry or behavioral change. Anxiety symptoms and functional impairments are independent of the actual panic attack. Panic attacks are uncued/ spontaneous. At least once of the attacks has been followed by 1 month (or more) of once (or more) of the following: * persistent concern about having additional attacks. * worry about the implications of the attack or its consequences (e.g losing control, having a heart attack, "going crazy") * a significant change in behavior related to the attacks The panic attacks are not due to the direct physiological effects of a drug or a medical condition. The panic attacks are not better accounted for by another mental disorder.

A young woman, age 21, is a talented sophomore at a prestigious college, where she lives in a dormitory. She describes herself as "shy" and "slow to warm up" in social situations. She used to drink one or two cans of beer on social occasions. She found that drinking alcohol reduced her anxiety, so she started taking her "social lubricant" before most social events. After a difficult breakup with her boyfriend about 15 months ago, she began drinking larger amounts of alcohol to "forget him". She has become irritable and impatient with friends and lost many of them. Her mother found empty bottles of hard liquor in her room during a visit and confronted her. The daughter exploded and erratic behavior and promised to cut back. Despite her efforts, she has found it difficult to reduce her drinking. Every night, she decides to drink only one beer but craves more and eventually ends up drinking a few shots as well and fails to do her schoolwork. She has been skipping her morning classes b/c she feels sick and has to have a drink to prevent shaking, sweating, and anxiety. She has also stopped hiking with friends on the weekends, b/c she worries that she may start shaking in the middle of a hike. Once, she fell off her bike while she was drunk and broke her wrist. Her grades have started to deteriorate. An instructor asked her to see a doctor after he became aware of her drinking behavior. What is the likely diagnosis? A. Adjustment disorder. B. Alcohol intoxication. C. Alcohol use disorder. D. Alcohol withdrawal.

C. Alcohol use disorder.

A 29- year- old graduate student is accompanied by her parents for an initial appointment with a psychiatrist. The student reports that her boyfriend broke up with her 2 months ago, and she continues to feel devastated. She immediately moved back into her parents' home for support. Her parents are used to this, she says, b/c "this happens all the time". Her parents help her with everything, including choosing her clothes each morning, washing her laundry, and making all of her meals. They recall, "Her boyfriends would always pay her bills and even choose her friends for her. She has never made it on her own." Despite their concerns, in many ways her parents like having her at home b/c she is always agreeable and respectful. She even volunteers to scrub all the bathrooms from top to bottom; she says, "Whatever I shave to do so they will let me stay there until I find a new boyfriend." What is the likely diagnosis? A. Avoidant personality disorder. B. Borderline personality disorder. C. Dependent personality disorder. D. Generalized anxiety disorder.

C. Dependent personality disorder. This disorder is characterized by an extreme need to be taken care of which leads to submissive behavior and clinginess as well as fears of seperation. It is more common in females.

A plastic surgeon refers a 22- year- old hairdresser with no known hx of mental illness to a psychiatrist. She underwent facial reconstruction and other surgeries after a disfiguring fire at her salon. When she was seen at a 1- year follow- up appointment, the exam room mirror alarmed her. She said that she "felt apart from" her body and was markedly distressed. She described the sensation as disabling b/c it occurs often at work. However, she is not able to clearly characterize her symptoms, aside from feeling "outside" herself. Aside from emotional distress from her symptoms, she has intact reality testing ability. What is the likely diagnosis? A. Acute stress disorder. B. Conversion disorder (functional neurological symptom disorder). C. Depersonalization/ derealization disorder. D. Dissociative amnesia with dissociative fugue.

C. Depersonalization/ derealization disorder. This disorder is characterized by episodes of Depersonalization (I.e sense of being detached from one's body or mental processes) and/ or derealization (feelings of unreality or being detached from the environment) while reality testing remains intact.

Police bring a woman to the hospital after they find her in an alley behind a restaurant. She appears to be in her late 20s. She cannot say who she is or where she lives. When asked what she was doing earlier, she recounts a reasonable story about having dinner at a restaurant and then having a few drinks. However, sh looks worn, and though she has been living on the streets and exposed to the elements for hours or days. She is negative for alcohol and drugs on laboratory testing. Aside from mild dehydration, she is medically stable. Physical exam shows no neurological abnormalities. Finally, staff members find in her belongings an employee identification card from a mall where a shooting occurred 2 days ealier. When asked about that event, she has no recollection of it and seem unbothered. Her speech is clear and fluent, and she denies any perceptual changes. Police match her to a missing person report that her mother placed the previous day. What is the likely diagnosis? A. Bipolar I disorder. B. Depersonalization/ derealization disorder. C. Dissociative amnesia with dissociative fugue. D. Schizophrenia.

C. Dissociative amnesia with dissociative fugue. This disorder is characterized by an inability to remember important information of a personal nature (typically associated with a traumatic or stressful event) on one or more occassions. Symptoms cause clinically significant impairment or distress. When there is purposeful travel or bewildered wandering that is associated with amnesia or other important personal information, then spicy "with dissociative fugue".

A 55- year- old woman presents to the hospital for treatment of acute myeloid leukemia, which was diagnosed 1 month ago. Her husband passed away 1 year ago from a cancer that he had battled for 3 years. She reports seeing a therapist for the "past couple of years" while dealing wit her husband's illness and death. Her primary care team noted that she is frequently tearful in the hospital and looked relieved when a psychiatric evaluation was suggested. On interview, she reports feeling sad much of the day but that she is trying to stay upbeat about her treatment. She also reports feelings of hopelessness. Her appetite and sleep have been poor, and she has lost 50 pounds since the death of her husband. She feels guilty- "I wish I had taken better care of my husband in the end". She reports feeling isolated from friends and family as well. She makes a vague complaint of anxiety, which "comes and goes" throughout the day and is being treated with a round- the- clock schedule of benzodiazepines. She says that she "does not care if cancer kills her too". What is the likely diagnosis? A. Adjustment disorder. B. Generalized anxiety disorder. C. Major depressive disorder. D. Uncomplicated bereavement.

C. Major depressive disorder This disorder involves a minimum of one major depressive episode can causes clinically significant distress or impairment.

A 55- year- old woman is referred to psychiatric consultation. She has a hx of HIV, contracted from a former sexual partner. She has been prescribed highly active antiretroviral therapy but for years has been only intermittently adherent. Currently, her CD4 count is 200 and her HIV viral load has been escalating. She complains that over the past several months she has been less mentally agile than in the past. She says it takes her significantly longer to rad the newspaper each morning. When she wants to communicate with her daughter, concentrating on writing a text message is difficult and she even finds "my fingers do not cooperate with me when I try to type". Her fine motor skills appear to be worsening. The woman denies any other physical symptoms, such as fever or headache. There are no focal findings on neurological exam. Head compute red tomography (CT), magnetic resonance imaging (MRI), electroencephalography, lumbar puncture, non- HIV viral polymerase chain reaction testing, antibody screens, metabolic parameters including hepatic and renal function, and urine toxicology are all negative. Neurospychological testing shows impairment in information processing speeds, motor skills, and attention. What is the likely diagnosis? A. Cryptococcal meningitis. B. Major depressive disorder. C. Mild neurocognitive disorder due to HIV infection. D. Vitamins B 12 deficiency.

C. Mild neurocognitive disorder due to HIV infection.

Which of the following has been eliminated from the criteria for the diagnosis of schizophrenia in DSM-5 in comparison with earlier versions of DSM? A. The duration criteria. B. The negative symptom criteria. C. The paranoid subtype. D. The requirement for a delusion, hallucination, or disorganized speech.

C. The paranoid subtype.

An 8- year- old boy wets the bed approximately 2 nights a week. He has been invited to weeklong summer sleep- away camp but feel afraid to go b/c of his bed- wetting. He is particularly upset b/c he did not have this problem last year. He feels a little less embarrassed when he finds out that his dad had the same problem when he was a boy. His father moved out of the home last year after marital separation. The boy does not take any medications and has no known medical problems. What is the likely diagnosis? A. Encopresis. B. Enuresis, diurnal only. C. Enuresis, nocturnal and diurnal. D. Enuresis, nocturnal only.

D. Enuresis, nocturnal only. This disorder is characterized by pattern of urinating in bed or clothing by an individual who is at least 5 years of age. Urination must occur at least twice a week over at least three consecutive months or must result in significant distress or impairment. Urination may be voluntary or involuntary. There are three subtypes: nocturnal only (during nighttime), diurnal only (during waking hours), and nocturnal and diurnal (waking hours and nighttime).

A gynecologist refers a 32- year- old hospital employee to a psychiatry clinic, noting that the pt requested testosterone shots and also a surgical referral. From the past medical records, the psychiatrist learns that the pt has normal female development and anatomy with no known abnormalities. The pt wears bland- colored baggy clothing, has very short hair, and goes by the name "Jo". She is initially brief with her answers but, after some time, is tearful, explaining that she has never been comfortable as a woman and is positive she is "supposed" to be a man. She feels ostracized at work and isolated socially. She has feelings of low self- worth and some depressed mood. What is the likely diagnosis? A. Adjustment disorder. B. Anorexia nervosa. C. Depersonalization/ derealization disorder D. Gender dysphoria in adolescents and adults.

D. Gender dysphoria in adolescents and adults. This disorder is a marked incongruence b/w one's experience/ expressed gender and assigned gender. There is a strong desire to be rid of one's primary and/ or secondary sex characteristics. There is a strong desire to be the other gender and to be treated as the other gender. The individual has a strong conviction that he/ she has typical feelings and reactions of the other gender. The symptoms need to be presents for at least 6 months and cause clinically significant distress or impairment.

A woman brings her 73- year- old father to the hospital b/c she is extremely concerned that he has virtually no memory of the day's events. He cannot recall where he has been or what he has done. Which additional piece of information would be more consistent with a diagnosis of dissociative amnesia than neurocognitive disorders? A. He cannot name the current president. B. He has a hand tremor and slowing on rapid- alternating- movements testing. C. He has extensive hx of chronic alcohol use. D. He was the victim of a mugging that morning.

D. He was the victim of a mugging that morning.

Which state to is correct regarding the bipolar disorders diagnostic class of DSM-5 in comparison with earlier versions of DSM? A. Anxious distress has been eliminated as a feature of bipolar disorder. B. Bipolar disorders have been consolidated under the diagnostic class of depressive disorders. C. Cyclothymia has been elimanated. D. Increased activity or energy has been added as a new core mood elevation symptom.

D. Increased activity or energy has been added as a new core mood elevation symptom.

What term best describes a person's mental status when he or she is responsive to external stimuli but is unable to stay awake? A. Agitated. B. Comatose. C. Delirious. D. Lethargic.

D. Lethargic. Lethargy- a state of decreased mental activity, characterized by sluggishness, drowsiness, inactivity, and reduced alertness.

A 5- year- old child believes that his thoughts, words, and actions cause specific outcomes in nature, and these outcomes defy what are the commonly understood laws of cause and effect. What is this is this phenomenon called? A. Grandiosity. B. Hallucination. C. Illusion. D. Magical thinking.

D. Magical thinking

A 50- year- old woman presents to the local hospital complaint of "a drug problem". She says that for the past 3 years her use of cocaine has been controllable, to the point that she has lost her housing and her job. She reports using increasing amounts of cocaine over the past year to "reach the same high". When she stops using cocaine, she experiences intense symptoms of anxiety, depression, fatigue, and nausea. She has tried multiple times to quit using cocaine altogether, but her efforts have been unsuccessful. She suffered a transient stroke once during a cocaine binge. She denies any mood problems or hallucinations prior to using substances. What is the likely diagnosis? A. Bipolar I disorder. B. Cocaine intoxication, without perceptual disturbances. C. Cocaine withdrawal. D. Moderate cocaine use disorder.

D. Moderate cocaine use disorder.

A 25- year- old man with no medical or psychiatric hx other than anemia travels frequently for his job. He has trouble on long flights b/c he feels the need to move his legs about and unfasten his seat belt to do so. When he is not permitted to get up, especially on "red eye" flights, he feels symmetrical burning and tingling in his legs. He is also very exhausted and notes that his sleep is not restful. He takes no medications except a daily multivitamin. What is the likely diagnosis? A. Adjustment disorder. B. Akathisia. C. Parkinson's disease. D. Restless legs syndrome.

D. Restless legs syndrome. Is a sensorimotor, neurological sleep disorder characterized by a desire to move the legs (or arms) associated with uncomfortable sensations (e.g creeping, crawling, tingling, burning or itching). The frequent movements are in response to the uncomfortable sensations. The symptoms must at least 3 times per week over a period of 3 months. It reduces sleep time and sleep disturbance. The syndrome is more common in women than men.

A 12- year- old boy feels tired of "standing out so much" at school. Several times a day he feels an uncomfortable sensation in his throat and then feels he "has to" cough or grunt, which temporarily makes the uncomfortable sensation go away. He also repeatedly shrugs his shoulders and stomps his feet at what appear to be random moments throughout the day. The other students no longer make fun of him the way the did when they were younger, but he still feels demoralized b/c he just wants to fit in with them. What is the likely diagnosis? A. Obsessive- compulsive disorder. B. Obsessive- compulsive personality disorder. C. Persistent motor or vocal tic disorder. D. Tourette's disorder.

D. Tourette's disorder. Involves both motor tics and vocal tics over the course of the disorder. Onset is before age 18. Tics wax and wane in frequency, but have persisted for more than 12 months.

A mother has learned during a conference with her 6- year- old son's teacher that her son tends to blurt out answers when students are asked to raise their hands. Also, he often intrudes on other children's activities during playtime, which has caused some disputes with his classmates. The mother notes that he often forgets to bring home his schoolwork. In addition, he often willingly starts a chore around the house, such as cleaning his room, but often switches to another activity and fails to finish the original task. What is the likely diagnosis? A. Attention- deficit/ hyperactivity disorder. B. Obsessive- compulsive disorder. C. Oppositional defiant disorder. D. Tourette's disorder.

A. Attention- deficit/ hyperactivity disorder. This disorder has a clear pattern of in attention and/ or hyperactivity not accounted for by an individual's developmental stage. ADHD is more common in males. Some symptoms are evident before age 12 and impairment must be significant and evident in at least two different settings. In preschool children, hyperactivity is the main manifestation. In school age children, in attention is more prominent. In adolescents, hyperactivity is less common and may be more related to figetiness, inner restlessness or impatience. In adulthood, impulsivity may remain problematic even when hyperactivity has diminished. In children, there must be at least 6 symptoms in either or both in attention and hyperactivity- impulsivity. In adolescents and adults (over age 17) the individual must present with five symptoms.

A primary care doctor refers a 46- year- old woman with many physical complaints for psychological assessment after extensive medical testing reveals no evidence of physical issues. The psychologists learns that the pt has an extensive hx of bodily complaints without medical basis. Her physical symptoms have been multi systemic and do not resemble the constellation of symptoms associated with specific medical illnesses. The referring doctor also reports that the pt has frustrated many primary care doctors in the past and has transitioned from one to another. The pt expresses her distress to the psychologist. She says, "None of this is in my head. It is real. I have talked to so many doctors and those doctors just can't figure it out. I have no answers, and I feel worse and worse." What is the likely diagnosis? A. Body dysmorphic disorder. B. Conversion disorder (functional neurological symptom disorder). C. Factitious disorder. D. Somatic symptom disorder.

D. Somatic symptom disorder. This disorder is characterized by six or more months of a general preoccupation with fears of having a serious disease that has not been detected based on the individual's misinterpretation of bodily symptoms (previously known as hypchondriasis). The conviction persists despite negative physical and laboratory findings. It causes clinically significant distress and impairment.

A 34- month- old boy rarely initiates eye contact with his mother and does not seem to engage in play with other children in his peer group at day care. At times, he behaves aggressively toward the other children. He is only minimally communicative with speech but, when vocal, is prone to repetitively use only single words. He gets very upset when his mother varies his morning clothing routine, always insisting, for example, that she put on his right sock first, before any other piece of clothing. What is the likely diagnosis? A. Autism spectrum disorder. B.. Global developmental delay. C. Intellectual disability (intellectual developmental delay disorder). D. Major depressive disorder.

A. Autism spectrum disorder. This is a new disorder under the DSM-5. It consolidates the following DSM-IV disorders: Autistic Disorders, Asperger's Disorder, Rett's Disorder, Childhood Disintegrative Disorder and Pervasive Developmental Disorder NOS. Symptoms of the disorder are represented in terms of severity in social communication/ interaction and restrictive repetitive behavior's/ interests/ activities. Symptoms include: reduced sharing of emotion and a failure of normal back and forth conversation; stereotyped or repetitive motor movements or speech; insistence of sameness: no flexibility in routines, ritualized patterns of verbal or nonverbal behavior, fixated interests or focus; and hyper or hypo sensitivity to sensory input. Severity is assessed separately for social communication and restricted, repetitive patterns of behavior.

A mother reports that her son, who is now in first grade, has not seemed to fit in with his classmates since he was in kindergarten. He has a difficult time fully engaging in conversations with them. Instead, he seem to talk "at" them. For example, in a monotone voice he persistently shares elaborate and detailed information about his favorite subject, classic cars, even when the other children are very disinterested. Although this child met all motor milestones appropriately, he often exhibits a repetitive behavior of stacking his cars in a line. He does not have auditory problems. What is the likely diagnosis? A. Autism spectrum disorder. B. Global developmental delay. C. language disorder. D. Social anxiety disorder (social phobia).

A. Autism spectrum disorder. This is a new disorder under the DSM-5. It consolidates the following DSM-IV disorders: Autistic Disorders, Asperger's Disorder, Rett's Disorder, Childhood Disintegrative Disorder and Pervasive Developmental Disorder NOS. Symptoms of the disorder are represented in terms of severity in social communication/ interaction and restrictive repetitive behavior's/ interests/ activities. Symptoms include: reduced sharing of emotion and a failure of normal back and forth conversation; stereotyped or repetitive motor movements or speech; insistence of sameness: no flexibility in routines, ritualized patterns of verbal or nonverbal behavior, fixated interests or focus; and hyper or hypo sensitivity to sensory input. Severity is assessed separately for social communication and restricted, repetitive patterns of behavior.

A man brings his 35- year- old wife, who has a hx of previous psychiatric hospitalizations, to the emergency department b/c of her unusual behavior. He states that she just spent $3,000 on a shopping spree.. She speaks animatedly and loudly: "I just remembered all these things I wanted to by!" She looks down at her hand and says, "I need this ring on my finger for good luck." She has been sleeping 5 hours per night, and her days are "packed with events, booked from morning to evening!" She describes her mood as "stressed" and her affect is labile. Her mental status examination is notable for a slender, well- groomed woman with meticulous makeup and good eye contact. She is cooperative on interview, but is difficult to interrupt. Her thought process is occasionally tangential, and her speech is fluent, with rapid rate. She denies any hallucinations. There is no evidence of psychomotor abnormalities or neurological deficits. Her urine toxicology screen is negative for illicit substances. What is the likely diagnosis? A. Bipolar I disorder. B. Borderline personality disorder. C. Obsessive- compulsive disorder. D. Schizophrenia.

A. Bipolar I disorder. This disorder involves a minimum of one manic episode with major depressive episodes (although mania may be the first manifestation of the disorder). In coding, indicate the type of episode the person is currently experiencing (or last experienced).

A man with schizophrenia manifests a failure to express feelings either verbally or nonverbally, even when talking about issues that would be expected to engage the emotions. His expressive gestures are very limited, and there is little animation is his facial expression or vocal inflections. Which term best describes the man's affect? A. Blunt. B. Depressed. C. Inappropriate. D. Labile.

A. Blunt.

A 47- year- old police officer presents in an outpatient clinic for evaluation of "weird experience". He says he was out of my mind, really paranoid" for a full 2 weeks, but then completely recovered. His symptoms began shortly after one of his colleagues was inexplicably shot during a routine traffic stop. Although everyone in the department has been upset by the event, the pt found himself increasingly preoccupied with what he called "mysterious signs and circumstances" around the time of the tragedy and what he felt was the police chief's desire to "sweep it all under the carpet". Two days after the event, he began to hear voices on his car radio warning him about surveillance by "corrupt" members of the police department, who were "trying to bring him over" to their side with "brainwashing KGB techniques". Soon after, he began to hear a "whine" from his earpiece, which he recognized as the brainwashing. Unable to bear it any longer, he crushed the earpiece. Although he was able to pass this off an an accident at the time, he remains terrified that he might "lose it" agin. Subsequently, he returned to his usual state and did not experience further psychotic symptoms. What is the likely diagnosis? A. Brief psychotic disorder. B. Delusional disorder, persecutory type. C. Schizophrenia. D. Schizophreniform disorder.

A. Brief psychotic disorder. This disorder is characterized by schizophrenic symptoms that have been evident at least 1 day and no more than 1 month. A SW should specify With Marked Stressor(s) if symptoms seem to be caused by a stressful event; Without Marked Stressor(s) if symptoms do not seem to be caused by a stressful event; With Postpartum Onset if symptoms appear within 4- weeks postpartum; specify fight Catatonia if the individual meets the criteria for Catatonia..

Match each term with the correct definition (each term may be used once, more than once, or not at all): A. Comulsion. B. Obsession. C. Phobia. D. Somatization. _ Repeated behavior that a person feels driven to perform. _ Repeated expression of psychological issues through physical symptoms, concerns, or complaints. _ Repeated, irrational, and enduring fear of an object or situation. _ Repeated though or image that provokes anxiety.

A. Compulsion- Repeated behavior that a person feels driven to perform. D. Somatization- Repeated expression of psychological issues through physical symptoms, concerns, or complaints. C. Phobia- Repeated, irrational, and enduring fear of an object or situation. B. Obsession- Repeated through the of image that provokes anxiety.

A woman takes her 32- year- old roommate, who has a hx of migraines, to the hospital after the roommate experienced sudden- onset blindness in both eyes. Her last migraine was 1 month ago, and she was not experiencing any headache symptoms at the time of exam. The preliminary visual exam revealed no blink to visual threat to either eye. The woman was unable to describe colors or identity people standing near the bed. Her CT and MRI were normal, and a cerebral vascular accident was ruled out. Ophthalmological exam revealed no abnormal findings. The blindness persisted for 3 days, during which time she was hospitalized. Further discussion with medical providers revealed that her boyfriend broke up with her the night before admission. What is the likely diagnosis? A. Conversion disorder (functional neurological symptom disorder). B. Dissociative identity disorder. C. Major depressive disorder. D. Somatic symptom disorder.

A. Conversion disorder (functional neurological symptom disorder). This disorder is characterized by loss of functioning in voluntary motor and sensory functions deemed not to be fully the result of a medical condition, a substance, or a phenomenon sanctioned by a culture. The loss of functioning is initiated or exacerbated by a stressful event is not caused exclusively by pain or sexual dysfunction. The disorder is more common in women than in men. The longer the duration of the individual's symptoms and the more regressed the individual is, the more difficult is the treatment.

Police bring a 30- year- old homeless man to the psychiatric emergency department after he called 911 b/c he thought people were controlling his mind through "microwaves". He has no previous psychiatric or medical hx, and his urine toxicology screen is negative for illicit substances. On interview, he appears very guarded, with limited cooperation, b/c he feels that he does not belong on a psychiatric unit. "I am not the problem", he says. "These microwaves are all around us, even if you cannot feel them." He denies any mood symptoms. Until a few years ago he was working full time, but he refuses to divulge his occupation "b/c I do not trust anyone". On examination, his speech is fluent with normal volume and rate, but his thought process is occasionally disorganized. His affect is blunted, and no psychomotor abnormalities are noted. He denies any hallucinations, but during the interview he occasionally stares behind the interviewer very intently, without responding to any questions. Which term best describes the man's experience of "microwaves"? A. Delusion. B. Hallucination. C. Ideas of reference. D. Illusion.

A. Delusion. A false belief based on incorrect inference about external reality that is firmly held despite what almost everyone else believes and despite what constitutes incontrovertible and obvious proof or evidence to the contrary. The belief is not ordinarily accepted by other members of the person's culture or subculture (I.e it is not an article of religious faith). When a false belief involves a value judgement, it is regarded as a delusion only when the judgement is so extreme as to defy credibility. Delusional conviction can sometimes be inferred from an overvalued idea (in which case the individual has an unreasonable belief or idea but does not hold it as firmly as is the case with a delusion). Delusions are subdivided according to their content.

A 21- year- old college student complains that he has developed "an uncomfortable relationship with food". His current symptoms seem to be consistent with an eating disorder. Which of the following pieces of information would help distinguish b/w anorexia nervosa and bulimia nervosa? A. He has lost significant weight and appears emaciated. B. He is highly critical of his body. C. He regularly binges on high- calorie, high- fat foods. D. He uses large quantities of laxative tablets each day.

A. He has lost significant weight and appears emaciated. The key difference b/w bulimia nervosa and anorexia nervosa, binge- eating/ purging type, is the current body weight. It is common for individuals to move from one diagnosis to the other depending on body weight, so evaluating current body weight and behavior is important.

For which DSM-5 diagnosis is there a greater prevalence in women than men? A. Alcohol use disorder. B. Bipolar I disorder. C. Major depressive disorder. D. Schizophrenia.

C. Major depressive disorder.

Match each term with the correct definition (each term may be used once, more than once, or not at all): A. Anhedonia B. Anorexia. C. Cataplexy. D. Catatonia. _ Loss of appetitie. _ Loss of interest of pleasure. _ Loss of muscle tone, with weakness. _ Loss of response to outside stimuli and muscular rididity.

B. Anorexia- Loss of appetite. A. Anhedonia- Loss of interest or pleasure. C. Cataplexy- Loss of muscle tone, with weakness. D. Catatonia- Loss of response to outside stimuli and muscular rigidity.

A 30- year- old retail manager at a cosmetics store with no previous psychiatric hx finds herself experiencing distinct periods of intense anxiety two to three times per day. She is concerned b/c she has never experienced anything like this in the past. During the episodes she feels tremulous, hot, and faint, with palpitations and fear of loss of control. She denies any drug use and any recent stressors. This problem has been getting worse over the past few months and has affected her ability to function at work. She also has been losing weight and eating more. She is worried that her health has affected her appearance b/c her skin looks different, her hair is thinning, and she has swelling in her neck. What is the likely diagnosis? A. Agoraphobia. B. Anxiety disorder due to another medical condition. C. Generalized anxiety disorder. D. Social anxiety disorder (social phobia).

B. Anxiety disorder due to another medical condition.

A woman brings her 32- year- old husband to a psychiatrist for "unusual behavior". She says that for the past 5 days he has been cleaning the house extensively, often late into the night. He wakes up 2 hours earlier than usual the next morning but does not appear tired. He says he feels "very happy and productive- the best I have ever been!" His wife denies any dangerous behavior's at home and reports he is able to continue working at his current job, albeit more productively than before. She recalls that 6 months ago, he seemed very depressed, with loss of interest, poor sleep, low energy, and impaired concentration that lasted 1 month. The pt has not been hospitalized previously. On interview, he is pleasant and cooperative. His speech is pressured but redirectable. His thought process is linear, and he denies any hallucinations. His urine toxicology screen is negative. What is the likely diagnosis? A. Bipolar I disorder. B. Bipolar II disorder. C. Schizophrenia. D. Substance/ medication- induced bipolar and related disorder.

B. Bipolar II disorder. This disorder involves a minimum of one hypomanic episode and major depressive episode(s).

A 21- year- old college student is preoccupied with the appearance of her nose. She has a small, 0.2- cm soft tissue nodule from a biking accident many years ago, but the area is well healed. She feels it is a serious flaw on her face, however, and is thinking about plastic surgery to correct the defect. She is concerned about dating anyone as long as she has this nodule on her nose, although she continues to enjoy spending time with friends. She denies any pain or discomfort, and she has no concerns about her body weight, which is average for her height. She continues to bike the same route daily at school. What is the likely diagnosis? A. Anorexia. B. Body dysmorphic disorder. C. Posttraumatic stress disorder. D. Somatic symptoms disorder.

B. Body dysmorphic disorder. This disorder is characterized by a preoccupation with a slight or perceived physical defect judged by the individual to negatively affect his or her appearance.

A 33- year- old man arrives at the hospital by ambulance after his girlfriend called the police during a dispute. While arguing with his girlfriend, the pt had locked himself in a bathroom and swallowed a dozen tablets from the medicine cabinet. On evaluation, the pt is drowsy but responsive to voice. He has thin, liner, parallel scars on the extensor surface of his left wrist. He states that he wanted to show his girlfriend how much she was hurting him and they says, "She is probably going to leave me, just like everyone else has". What is the likely diagnosis? A. Adjustment disorder. B. Borderline personality disorder. C. Dependent personality disorder. D. Histrionic personality disorder.

B. Borderline personality disorder. This disorder is characterized by instability in relationships, self- image and affect, as well as impulsive behavior. It is more common in females.

On which of the following diagnostic frameworks are the DSM-5 and ICD classification systems based? A. Binary B. Categorical. C. Fuzzy. D. Probabilistic.

B. Categorical. Most of the criteria in DSM-5 are categorical; in other words, the person is rated as either having symptom or not have it.

A 50- year-old Iranian woman being treated for an ankle sprain casually reports having had a conversation with a deceased relative. The emergency department physician is concerned that she has had both auditory and visual hallucinations and asks the psychiatrist to rule out psychotic disorder. At the interview, the woman endorses having had, on multiple occassions, visits from her deceased mother, in which she has conversations with her and gets advice concerning psychosocial matters. She is not distressed by the incidents and, in fact, finds there experiences to be spiritual blessings. She does not exhibit any other symptoms of psychotic disorder. Her mental status examination is remarkable for a pleasant and cooperative demeanor, normal mood, and full range of affect. Her thought process in linear. During the interview, her husband corroborates her attitude that these are real spiritual experiences and wonders why a psychiatrist has been called to interview his wife. The Frasi- speaking interpreter says that the woman's story is common among Iranian immigrants. What is the likely diagnosis? A. Brief psychotic disorder. B. Culturally appropriate behavior. C. Schizophrenia. D. Shared psychotic disorder (folie a deux).

B. Culturally appropriate behavior.

A 56- year- old biologist has had a very successful career, publishing extensive research on the aging process. Now he believes he has found the key to eternal life. For the past several months, he has been quite secretive and refuses to come to work group meetings regarding upcoming projects. He has installed a dead bolt on his office at home and insists that extra security personnel be hired to guard his research lab. He refuses to present his material or discuss it with his colleagues or family, claiming the knowledge is too powerful to be made public. He is not having any visual or auditory hallucinations, and he is able to support his statements with logical arguments. He has a hx of depression. What is the likely diagnosis? A. Cocaine intoxication, without perceptual distrubances. B. Delusional disorder, grandiose type. C. Major depressive disorder with psychotic features. D. Schizophrenia.

B. Delusional disorder, grandiose type. This disorder is characterized by a minimum of one non- bizarre delusion and must be evident for a minimum of 1 month. It can be accompanied by tactile of olfactory hallucinations if related to the delusion. An individual's psychosocial functioning, apart from the delusions, is normal. Any concurrent mood episodes are brief relative to the duration of the delusions. Grandiose type- is diagnosed when the individual is convinced that he or she has a special talent or understanding of some phenomenon or that he or she has accomplished something of great importance.

A woman brings her 45- year- old husband to a neurologists for "memory and behavior" problems. She states that for the past several years, he has been alternately dependent or hostile toward her, as if he were two distinct individuals. When he is dependent, he follows her around everywhere, like a child. When he is hostile, he ignores her and will curse at her if she asks him to do household chores. He also tells her to call him by a different name. Each phase lasts for months at a time. He has forgotten important details of his childhood, such as the names of his parents and the town where he grew up. His wife denies that he uses any alcohol or other substances and denies that he has a hx of seizures or sleep disturbances. She has not noticed any impulsive or unsafe behavior's. On exam, the man is silent and stares blankly at the wall, without evidence of psychomotor abnormalities. He responds very briefly to questions but denies any hallucinations, changes in mood, or elevated anxiety. A thorough workup including CT and electoencephalography testing fails to reveal any clear neurological abnormalities. What is the likely diagnosis? A. Bipolar I disorder. B. Dissociative identity disorder. C. Major depressive disorder with psychotic features. D. Schizophrenia.

B. Dissociative identity disorder. This disorder is characterized by the presence of two distinct identities that alternately take control of the individual's behavior accompanied by extensive forgetting of personal information. In general, this disorder is the result of a traumatic experience(s) in which dissociative is a defense mechanism. The symptoms are not the result of a medical condition or a substance.

A 10- year- old boy who has severe developmental delay exhibits immediate and involuntary repetition and ambient sounds and vocalizations made by other people. What is the term for this phenomenon? A. Dysarthria B. Echolalia C. Echopraxia D. Word salad

B. Echolalia

A mother who recently moved to a new area brings her 6-year- old son for evaluation by a pediatrician. She worries that he is h aging diarrhea b/c she finds stool stains on his underwear. She is not sure how often he goes to the bathroom or the nature of his stools b/c for many years she has worked two or three job and has had to leave him in day care centers. On exam,, he is shy and avoidant of social interaction but appears to have no developmental delay with respect to cognition and language. The pediatrician notes a hard golf ball- sized structure on the pt's left abdomen on deep palpation. What is the likely diagnosis? A. Conduct disorder. B. Encopresis. C. Enuresis. D. Mental retardation.

B. Encopresis. This disorder is characterized by a pattern of expelling feces voluntarily or involuntarily into places deemed inappropriate (e.g clothing, floor) by an individual who is at least 4 years old. There must be a minimum of one of these events each month over at least three months and it must not be directly related to the effects of a substance or a medical condition. It is more common in males.

The hematology service requests a psychiatric consult for a 45- year- old woman with generalized bruising. The hematology team cannot find a cause for her physical findings, which had led to three hospital admissions in the past 5 months. The pt is a nurse at the hospital. On interview she is very tearful. Bruising is predominantly circumferential on her arms and legs, with none on her face or torso. She does have some venipunture marks on her abdomen, however, as well as a wristband from another hospital. She appreciates the consult but asks the psychiatrists to leave when her ex- husband comes for his daily visit to watch videos of their daughter. She explains that they lost their daughter to a chronic illness at age 8. What is the likely diagnosis? A. Conversion disorder (functional neurological symptom disorder). B. Factitious disorder. C. Malingering. D. Somatic symptom disorder.

B. Factitious disorder. This disorder is characterized by the repeated feigning, inducing, or aggravating physical and psychological symptoms motivated by a desire to receive medical care and be involved in the medial system, in the absence of financial or other external incentives. Sometimes referred to as Munchausen Syndrome, individuals embellish their personal hx, chronically fabricate symptoms to gain hospital admission and move from hospital to hosptial. The onset is typically in the 20s or 30s. This disorder occur more often in women than men.

A 35- year- old man tells his therapist that he is having marital problems b/c the object of his sexual arousal is his wife's underwear. He reports that he and his wife have sexual intercourse in a limited fashion, much less often than he prefers. He does not wear women's clothing, but he fulfills his sexual desires through masturbation while holding his wife's underwear or other women's undergarments that he has purchased. This activity is his long- standing pattern of sexual behavior and has been problematic in the relationship with his wife from early on. He reports that his wife has "caught" him masturbating with women's undergarments on multiple occasions and she things he is "weird". What is the likely diagnosis? A. Erectile disorder. B. Fetishistic disorder. C. Gender dysphoria in adolescents and adults. D. Travesties disorder.

B. Fetishistic disorder. This disorder involves intense and recurring urges, fantasies, and behaviors (minimum of 6 months) that are sexually arousing and are focused on inanimate objects. This behavior cannot be restricted to female clothing used with cross- dressing or to vibrators and similar devices and must result in marked distress or in significant impairment in functioning.

A 22- year- old college student presents to a MH clinic for an initial visit. She has not turned in any of her papers on time this term b/c she worries about getting poor grades, and this worry has affected her ability to concentrate. She is also very concerned about getting a job after she graduates, despite multiple meetings with a career counselor. She says that she is concerned with the sanitation at the local gym and prefers to to play recreational sports b/c she might be injured. She does not have any rituals or checking behavior's. She describes her mood as "irritable" and says that she feels "tense all the time". Her appetite has not changed, and she continues to enjoy watching movies. She denies low energy and has not had any thoughts of harming herself. She tried cocaine last year but has not used any illicit substances or alcohol recently. What is the likely diagnosis? A. Adjustment disorder. B. Generalized anxiety disorder. C. Major depressive disorder. D. Obsessive- compulsive disorder.

B. Generalized anxiety disorder. This disorder involves excessive anxiety and worry about a number of things that persists for a minimum of 6 months. The behavior is not due to a medical condition or substance.

A mother brings her 3- year- old daughter to the pediatrician b/c she "does not talk very much". Her vocabulary consists of only 10 words, and she speaks only one word at a time. Her past medical hx is notable for delays in developmental milestones: she walked independently at age 2, developed a pincer grasp at 15 months, and continues to have stranger anxiety. A hearing test completed in the clinic was normal. What is the likely diagnosis? A. Autism spectrum disorder. B. Global developmental delay. C. Selective mutism. D. Tic disorder.

B. Global developmental delay. Is reserved for children under the age of 5 when clinical severity cannot be reliably assessed through standardized testing. The child fails to meet expected developmental milestones in several areas of intellectual functioning. Reassessment after a period of time is required.

A 45- year- old man presents to his primary care doctor, complaining of chronic chest pain for the past year. He has seen multiple specialists, who tell him the pain is from uncomplicated acid reflux; however, he thinks he must have serious heart disease. He continues to worry about his heart despite reassurance from multiple cardiologists; he acknowledges that it is possible the cardiologists are correct. He denies hallucinations, and there is no evidence of delusional content. His concern about his heart has prevented him from vacationing with his family. He denies feeling depressed or anxious or having any problems with sleep or appetitie, and he continues to enjoy hobbies at home. What is the likely diagnosis? A. Generalized anxiety disorder. B. Illness anxiety disorder. C. Major depressive disorder. D. Panic disorder.

B. Illness anxiety disorder. This disorder is characterized by a preoccupation with being sick or with developing a disease. There are few or no somatic symptoms present, but the individual is primarily concerned with the idea that he or she is ill. The diagnosis can be used for individuals who do have a medical illness but whose anxiety is out of proportion to the diagnosis and who assume the worst possible outcome. The anxiety is incapacitating and causes emotional distress or impairs the individual's ability to function. Some individuals may visit physicians (care- seeking type) while others may not (care- avoidant type).

Match each mental disorder with the most accurate statement regarding prevalence by gender (each item may be use once, more than once, or not at all): A. More prevalent among men. B. More prevalent among women. C. Equally prevalent among men and women. _ Restless legs syndrome. _ Major depressive disorder. _ Generalized anxiety disorder.

B. More prevalent among women - Restless legs syndrome. B. More prevalent among women- Major depressive disorder. B. More prevalent among women- Generalized anxiety disorder

A mother brought her 12- year- old son to the clinic. He is a pleasant student who has developed difficulty staying awake in class for the past 6 months. He is very upset that no matter how much he tries, he suddenly becomes very sleepy, every during his favorite classes, and then falls asleep and naps. He feels refreshed after these naps but feels embarrassed when other students laughter at him or when teachers notice those episodes, which happen at least three time per week. His parents ensure that he goes to sleep at a regular time and sleeps for at least 8 hours, but this regularly has not helped with his lapses into sleep during the day. Recently, he started having episodes where he suddenly loses muscle tone and falls but remains conscious. these falls usually happen when he is excited or laughs. He has had a few such falls over the past month. Last time, it happened when he got very excited b/c his school's basketball team won the state championship, and he felt very embarrassed in front of his classmates. He has begun avoiding social events at school b/c he is worried he may have one of these falls. The extensive neurological and cardiac work ups were un revealing. The nocturnal sleep polysomnography shows rapid eye movement (REM) sleep latency of less than 15 minutes. What is the likely diagnosis? A. Circadian rhythm sleep- walk disorder. B. Narcolepsy. C. Non- rapid eye movement sleep arousal disorder. D. Rapid eye movement sleep behavior disorder.

B. Narcolepsy. This disorder is characterized by sudden, recurrent periods of irrepressible need to sleep, lapses into sleep or naps that occur 3 times per week for a minimum of 3 months. The individual experiences extreme drowsiness every 3-4 hours, sleep paralysis, loss of muscle tone (cataplexy) and sleep attacks. There exists the occurrence of components of REM sleep as the individual moves from sleep to wakefulness.

Concerned parents bring a 33- year- old man to the psychiatrist. He appears to be his stated age, with erythematous face and holding his reddened and chapped hands away from his body. He takes a few minutes to adjust into the seat. He appears very embarrassed during the evaluation as he shares his story. He describes being very concerned about getting sick from the germs in the environment. This concern translates into frequent hand washing and showering. He explains that he has a particular ritual for these grooming procedures and that if he feels that he made a mistake, he has to repeat the whole procedure from the start. He specks approximately 5 hours per day on his grooming rituals. He also shares that things in his room have to be in a particular order; otherwise, he feels very uncomfortable and thus spends significant time readjusting these items. Moreover, when driving, he frequently worries that the ran over someone, and he has to go back to make sure the no body is lying on the ground. This worry has led him to avoid driving. He realizes that his concerns are irrational but cannot stop his rituals b/c they help with the significant anxiety that accompanies his troubling thoughts. He has not worked for the past 5 years b/c his rituals and worries caused him to be late and inefficient at his job. He currently lives with and is supported by his parents. He finally agreed to come in for an evaluation at their urging. What is the likely diagnosis? A. Generalized anxiety disorder. B. Obsessive- compulsive disorder. C. Obsessive- compulsive personality disorder. D. Panic disorder.

B. Obsessive- compulsive disorder. This disorder is characterized by anxiety- producing obsessions (persistent thoughts, urges, or images) or compulsions (behavior's the individual feels compelled to do repeatedly according to rigid rules). The obsessions or compulsions are time- consuming or cause clinically significant distress or impairment in social, occupational or other important areas of functioning.

A 24- year- old woman experiences severe dysphoria and anhedonia affecting her ability to perform well at her job. Which of the following additional pieces of information would be more consistent with a diagnosis of major depressive disorder than a diagnosis of premenstrual dysphoric disorder? A. The pt has been feeling cognitively hazy, confused, and forgetful. B. The pt has had suicidal thoughts every day for the past 3 weeks, and yesterday she searched "how to kill yourself" on the Internet. C. The pt was very irritable and rejection sensitive for 4 days last week, followed by rapid resolution. D. The pt's symptoms have caused strain in her relationship with her boyfriend.

B. The pt has had suicidal thoughts every day for the past 3 weeks, and yesterday she searched "how to kill yourself" on the Internet.

Which statute net is correct regarding the World Health Organization Disability Assessment Schedule 2.0 included in DSM-5? A. The scale assesses only the three most important domains of hallucinations, delusions, and disorganized speech. B. The scale is based on symptoms experienced by the pt in the previous 30 days. C. The scale is most accurate if pts, rather than clinicians, provide the responses. D. The scale should only be completed if the clinician is certain of the symptom severity.

B. The scale is based on symptoms experienced by then pt in the previous 30 days. The WHODAS 2.0 is used to measure the degree of disability, secondary to a health or mental health condition, experienced in everyday life during the last 30 days.

Match each term with the correct definition (each term may be used once, more than once, or not at all): A. Agnosia. B. Anhedonia. C. Aphasia. D. Apraxia. _ Problems in comprehending language despite intact sensory function. _ Problems in performing purposeful motor activities despite intact motor function. _ Problems in producing language despite intact motor function. _ Problems in recognizing objects despite intact sensory function.

C. Aphasia- Problems in comprehending language despite intact sensory function. D. Apraxia- Problems in performing purposeful motor activities despite intact motor function. C. Aphasia- Problems in producing language despite intact motor function. A. Agnosia- Problems in recognizing objects despite intact sensory function. Aphasia- loss of impairment of the power to use or comprehend words usually resulting from brain damage. Apraxia- loss of impairment of the ability to execute complex coordinated movements without muscular or sensory impairment Agnosia- loss of ability to recognize objects, person's, sounds, shapes, or smells that occurs in the absence of either impairment of the specific sense or significant memory loss.

Match each description with the personality disorder for which it is most highly characteristic (each disorder may be used once, more than once, or not at all): A. Antisocial personality disorder B. Borderline personality disorder. C. Histrionic personality disorder. D. Narcissistic personality disorder. _ Consistently uses physical appearance to draw attention to onself. _ Failure to conform to social norms and without remorse. _ Grandiose sense of self- importance. _ Self- dramatization, theratrically, and exaggerated expression of emotion.

C. Histrionic personality disordder- Consistently uses physical appearance to draw attention to onself. A. Antisocial personality disorder - Failure to conform to social norms and without remorse. B. Borderline personality disorder- Frantic efforts to avoid real or imagined abandonment. D. Narcissistic personality disorder- Grandiose sense of self- importance. C. Histrionic personality disorder- Self- dramatization, theatricality, and exaggerated expression of emotion.

A 67- year- old man has been living by himself for demands in an apartment, but his daughter in concerned about his safety there. His rooms are stacked to the ceilings with items such as books and souvenirs, to the point that he cannot move around without shifting items. She worries that he may trip and fall in the apartment and has offered to help discard items; however, he finds it distressing to remove anything b/c he thinks everything will be of use to him someday. He denies any anxiety with his current living situation and prefers that it remain this way. What is the likely diagnosis? A. Delusional disorder. B. Generalized anxiety disorder. C. Hoarding disorder. D. Obsessive- compulsive personality disorder.

C. Hoarding disorder. This disorder is characterized by a persistent difficulty discarding or parting with possessions. The accumulation of possessions interferes with living areas of the home. The hoarding creates clinically significant distress and impairment.

A primary care physician refers a 45- year- old married man to a psychiatrist for evaluation of his anxiety. The pt explains that he worries that there is something seriously wrong within his abdomen but that his primary care doctor has not been able to identity it. He shares that he has had intermittent pain in his abdominal area. Although physical exam, laboratory work, endoscopy, imaging, and expert consultation have not revealed abnormalities, he is worried that his pain is a sign of cancer that is being missed. He relays lifelong concerns about his health. His wife shares that the is always hypervigilant about any signs of illness, and she embarrassingly chuckles that their primary care doctor "must be a very patient man" b/c her husband constantly e-mails and calls him with various concerns. She describes her husband as an overall healthy man. The pt denies feeling depressed but again stated how anxious he is that something serious is being missed. What is the likely diagnosis? A. Body dysmorphic disorder. B. Generalized anxiety disorder. C. Illness anxiety disoder. D. Somatic symptom disorder.

C. Illness anxiety disorder. This disorder is characterized by a preoccupation with being sick or with developing a disease. There a few or no somatic symptoms present, but the individual is primarily concerned with the idea that he or she is ill. The diagnosis can be used for individuals who do have a medical illness but whose anxiety is out of proportion to the diagnosis and who assume the worst possible outcome. The anxiety is incapacitating and causes emotional distress or impairs the individual's ability to function. Some individuals may visit physicians (care- seeking type) while others may not (care- avoidant type).

A 22- year- old woman is brought to the hospital after being found wandering on a school playground. She is not able to read or fill out basic paperwork. On exam, she is noted to be pursuing the picture books in the pediatric waiting area and laughing in a childlike manner. She is wearing a T-shirt, sweatpants, and shoes with hook-and-loop fasteners and has a wristband with her identification and her mother's contact information. The woman can say her name and address and answer simple yes/ no questions but cannot answer compound or complicated questions. She begins to cry after not being able to answer many questions. A review of her records indicates that she has not psychiatric hx but still sees her pediatrician, accompanied by her mother. Recent testing places her at an IQ of 50. She has no basic laboratory abnormalities and a negative urine toxicology screen. What is the likely diagnosis? A. Conduct disorder. B. Dissociative identity disorder. C. Intellectual disability (intellectual developmental disorder). D. Selective mutism.

C. Intellectual disability (intellectual developmental disorder). Intellectual disability replaces the term "mental retardation". It is a term that is more commonly used by the lay public, advocacy groups and federal statutes. Is characterized by both intellectual and adaptive functioning deficits that begin during the developmental period. Intellectual deficits occur in reasoning, problem solving, planning, abstract thinking, judgement, academics and an inability to learn from experience. Intellectual functioning deficits are manifested by clinical observation and standard intelligence testing. Adaptive functioning deficits are manifested in failure to meet developmental and sociocultural standards of personal independence and social responsiblity. There are limitations in activities of daily living, communication, independent living, and social interactions across multiple situations (e.g home, school, work).

Police arrest a 26- year- old man for smashing the windows of a coworker's car after an argument at work. The man's wife says the he has a hx of destroying property or items of value when provoked, in a manner that is "extreme" as compared to the magnitude of the inciting trigger or event. He denies any hx of head trauma, difficulty with attention, depressed or labile mood, restlessness, or any problems with sleep or appetite. When he is not provoked, he is calm and pleasant. He denies any alcohol or substance use, and his wife corroborates this information. There is no evidence of fidgeting or psychomotor agitation on exam. What is the likely diagnosis? A. Attention- deficit/ hyperactivity disorder. B. Bipolar I disorder. C. Intermittent explosive disorder. D. Panic disorder.

C. Intermittent explosive disorder. This disorder is characterized by aggressive outbursts that are rapid onset and short duration. The aggressiveness is clearly out of proportion to the precipitating factors. The episodes involve verbal assault, destructive/ nondestructive property assault, injurious/ non-injurious physical assault. Verbal episodes occur at least two times per week for a period of 3 months. Damage to property or physical assault must occur at least 3 times during a 12 month period. The disorder may appear as early as 6 years or equivalent developmental level. The average age of onset ranges from 13- 21 years of age.

A 29- year- old woman and her new boyfriend had an unplanned pregnancy culminating in a first- trimester miscarriage last year. She has always considered herself an anxious person but has felt even more anxious after the miscarriage. She became pregnant a second time, again unplanned, and she gave birth to a healthy baby boy after an emergency C- section delivery. Their son is now 4 months old. Her boyfriend has become emotionally distant. He moved out of their apartment a couple of weeks after the birth, stating, "I am not ready for this life." For the past several weeks, she has experienced escalating low mood, irritability, tearfulness, poor sleep, and constant fatigue. The pt reports that she is frightened by what she is thinking. She states, "Sometimes I feel I could care less about taking care of my son or, even worse, that I want to just physically push him way. It is really scary. How can I ever be a good mother to him? I am miserable. I do not know if I can take it anymore." Her first psychiatric contact was at age 22, when she was treated with citalopram for a depressive episode precipitated by being laid off from her first job after college. She says, "I have never quite been happy again since then. Life has been hard." What is the likely diagnosis? A. Bipolar II disorder, current episode depressed, with peripartum onset. B. Brief psychotic disorder, with peripartum onset. C. Major depressive disorder, with peripartum onset. D. Persistent depressive disorder.

C. Major depressive disorder, with peripartum onset. This disorder involves a minimum of one major depressive episode and causes clinically significant distress or impairment.

An 18- year- old single mother presents to the clinic after 6 weeks of low mood and trouble sleeping. At first, she attributed her poor sleep to getting up every 2 hours to feed her 8- week- old daughter, but more recently her mother has been caring for the baby at night. Despite this, the pt reports continuing poor sleep, with low energy and poor concentration. She is less interested in leaving the house and seeing others and less excited about playing with her daughter. She is not very interested in eating but pushes herself to eat, "because I want to make good milk for the baby". She denies wanting to harm herself or others and feels that, with her mother's help, she is able to care for the baby well. What is the likely diagnosis? A. Bipolar I disorder, current episode depressed, with peripartum onset. B. Major depressive disorder, unspecified. C. Major depressive disorder, with peripartum onset. D. Persistent depressive disorder.

C. Major depressive disorder, with peripartum onset. This disorder involves a minimum of one major depressive episode and causes clinically significant distress or impairment.

A 76- year- old widowed, retired hx professor comes to the clinic with his daughter to seek help for his memory difficulties, which started about 3 years ago and have progressed gradually. He stopped driving 6 months ago after being involved in a car accident. He recently moved in with his daughter, and she has been taking him to appointments, managing his finances, and attending to his care. A recent brain CT scan shows cortical atrophy and ventricular enlargement. His thyroid- stimulating hormone and B12 levels are within normal limits. His daughter starts crying while reporting that he has had many verbal disagreements with her. Lately has been accusing her of controlling him and becomes very agitated. What is the likely diagnosis? A. Delirium. B. Intermittent explosive disorder. C. Major neurocognitive disorder due to Alzheimer's disease. D. Major neurocognitive disorder with Lewy bodies.

C. Major neurocognitive disorder due to Alzheimer's disease.

A young man comes to the clinic. He is a 26- year- old veteran who previously was involved in combat in the Middle East, where he was exposed to a few explosions. Since he returned about 4 months ago, he has had difficulty sleeping and has experienced frequent nightmares. He has avoided going out, b/c any loud noise startles him, disturbed him significantly, and brings back memories of the explosions. He sees himself as a failure and believes that he was not strong enough to tolerate combat. When his friends have approached him, their comments and jokes irritate him, and he isolates himself more. He has not been able to concentrate well and failed to pass the online courses he has been taking for his college degree. His mother has become increasingly concerned and encouraged him to come to the clinic. What is the likely diagnosis? A. Acute stress disorder. B. Adjustment disorder. C. Posttraumatic stress disorder. D. Social anxiety disorder.

C. Posttraumatic stress disorder. This disorder is defined as exposure to actual or threatened death, serious injury, or sexual violence in one or more of the following ways: directly experiencing a traumatic event, witnessing the event that occurred to others, learning that the traumatic event occurred to a close family member or close friend, experiencing repeated or extreme exposure to aversive details of the traumatic events. The symptoms have occurred for over a 1- month period. The individual has increased arousal and avoids internal and external stimuli that are reminiscent of the trauma. There are specific criteria for children 6 years and younger.

A woman brings her 26- year- old boyfriend, an army veteran, to a psychiatric clinic after his primary care physician refers him for evaluation of "stress". She says that after her boyfriend's return from Iraq, they moved in together, but they already need to sleep in separate beds after two episodes in which he punched her while she was sleeping. The events both occurred in the early morning and were so violent that she has bruises on her neck and arms. The man looks extremely distraught. He says he frequently dreams that he is back in the Middle East, in hand- to- hand combat, and is afraid that these dreams will cause his girlfriend to leave him. Her friends already say that he is a "crazy vet" who will "end up killing her". The thought of this is deeply painful to him, b/c they have a supportive, caring relationships and have spoken about getting married and starting a family. A review of systems is negative, except for increasing insomnia (delayed sleep onset). A thorough hx and collaterals information indicate that he does not meet criteria for Posttraumatic stress disorder. He works as a vendor at the local baseball stadium, a job that he says he enjoys, but he is afraid that his poor sleep is beginning to affect her performance at work. What is the likely diagnosis? A. Bipolar I disorder, current episodic manic. B. Generalized anxiety disorder. C. REM sleep behavior disorder. D. Schzophrenia.

C. REM sleep behavior disorder. Is characterized by repeated episodes or arousal during sleep associated with vocalization and/ or complex motor behavior's during REM sleep. The behavior's reflect action- filled or violent dreams and can result in significant injury to the individual or bed partner. Upon awakening the individual if fully alert and can usually recall the dream content.

A 3- year- old girls in a park attempts to sit on the lap of a homeless person. She is pulled away by her mother who recently has been out of town. While the mother was away, the girl had been under the care of the grandmother who spent little time with the girl. The girl does not respond to the affection shown to her by her mother or grandmother. The girl is indiscriminately affectionate with strangers. She has no evidence of motor abnormalities and has met all developmental milestones appropriately. What is the likely diagnosis? A. Autism spectrum disorder. B. Mental retardation. C. Reactive attachment disorder. D. Rett syndrome.

C. Reactive attachment disorder. This disorder is characterized as a disruption in a child's normal attachment behavior. It is the result of grossly negligent parenting and maltreatment. The child exhibits a pattern of inhibited, emotionally withdrawn behavior toward adult caregivers. There is minimal social and emotional responsiveness to others, episodes of unexplained irritability, sadness or fearfulness with adult caregivers. The onset must have been before age 5. Developmentally the child should be at least 9 months old.

A man brings her 34- year- old girlfriend to the emergency department for worsening depression and suicidal thoughts. She reports feeling more depressed and isolated from her friends and family during the past month. She worries about being watched by her neighbors and thus has been avoiding going outside. She has difficulty falling asleep at night and has poor energy during the day. She has not showered in 5 days. She admits that she has been hearing voices that make derogatory statements about her. She had several previous depressive episodes and two psychiatric hospitalizations with similar presentations. Which piece of information would help diagnose Schizoaffective disorder in this woman? A. She has auditory hallucinations that are present only in the context of her mood symptoms. B. She has had one manic epidode. C. She has been hearing voices for weeks without depressed or elevated mood. D. She has heard similar voices before when she was depressed.

C. She has been hearing voices for weeks without depressed or elevated mood.

A 27- year- old man presents to a psychiatrist for evaluation of anxiety. He explains that he recently got a promotion at his job, which requires frequent traveling. Although he is excited about his new role, he is terrified about the prospect of flying. He describes that the last time he flew, 5 years ago, he felt extremely anxious, with rapid heart rate, rapid breathing, nausea, and tingling in his hands. He was terrified that the plane would crash and he would die. He has avoided flying ever since, but he can also feel extremely anxious to the point of hyperventilating when even a discussion of flying comes up. Now he has a business trip in 3 weeks and he is "mortified". He is considering whether to ask for a demotion. He denies feeling anxious in any other situations. What is the likely diagnosis? A. Generalized anxiety disorder. B. Panic disorder. C. Simple phobia. D. Social anxiety disorder.

C. Simple phobia.

A mother brings her 11- year- old daughter for assessment. The girl's teachers are uncertain whether she should advance to the next grade level. She enjoys doing math problems and is always enthusiastic when doing experiments in the science lab. However, in English and hx class, she seems like a very different student. She sits in the back of the classroom, looking distracted, and avoids reading out loud when asked. When she does read aloud, her reading lacks fluidity and is often peppered with inaccuracies. Her reading comprehension test results show that she is well below grade- level expectation. She does not make spelling mistakes in her homework assignments. What is the likely diagnosis? A. Specific learning disorder. B. Specific learning disorder, with impairment in mathematics. C. Specific learning disorder ,with impairment in reading. D. Specific learning disorder, with impairment in written expression.

C. Specific learning disorder, with impairment in reading. This disorder involves a persistent inability to learn or use academic skills as quickly or as accurately as other children of the same developmental age. The specific learning difficulties occur during school- age years and may not manifest until the demands of school exceed the individual's abilities. The disorder significantly impairs the person's ability to perform activities of daily living or In academic areas. This disorder tends to be more common in males.

Each week a 52- year- old woman bakes bread using the recipe and techniques taught to her by her grandmother. She describes this behavior as an important "ritual" in her life. When does this behavior become a symptom that would appropriately be considered a part of a diagnostic assessement? A. At all times this behavior should be considered a symptom. B. At no time should this behavior be considered a symptom. C. When engaging in this behavior is no longer of interest or is distressing to her. D. When engaging in this behavior is very time consuming for her.

C. When engaging in this behavior is no longer of interest or is distressing to her.

A 66- year- old moderately obese man has a long hx of hypertension. He is seen by his primary care physician for a follow- up appointment after his second stroke, which happened about 2 months ago. He reports that the weakness in his right leg is improving with physical therapy, but he has had difficulty finding his way from the rehabilitation center, which is only a few blocks from his home. His wife reports that her husband's first stroke, which happened about 2 years ago, he started having memory diffiulties. The man used to enjoy playing chess with his grandchildren, but his memory deteriorated so significantly after his second stroke that he is unable to play and even forgets their names frequently. The man is pleasant, articulates words well, but shows significant difficulty finding words. He is not oriented to time, performs poorly in calculatioin, and is unable to name familiar objects, such as a pen or watch. While the clinician reviews the man's medications, the man's show significant difficulty remembering the medications' names and instructions. He appears frustrated and ask for a pen to write down the instructions. What is the likely diagnosis? A. Delirium. B. Major. Neurocognitive disorder due to Alzheimer's disease. C. Major neurocognitive disorder due to another medical condition. D. Major vascular neurocognitive disorder.

D. Major vascular neurocognitive disorder.

A doctor sees a middle- aged homeless man who was admitted to the medical unit 24 hours earlier for a workup of heart failure. The man has received appropriate medication for his heart but reports puzzling symptoms unrelated to his cardiac condition, such as feeling very dysphoric, with nausea and vomiting, muscle aches, and insomnia overnight. On physical exam,, the man is sweating profusely, and his pupils are dilated. He keeps asking the nurse for "morphine for pain". At admission he was positive for opioids. What is the likely diagnosis? A. Alcohol withdrawal. B. Factitious disorder. C. Opioid intoxication. D. Opioid withdrawal.

D. Opioid withdrawal.

A 12- year- old girl persistently misbehaves at home. Recently she called 911, reporting that her parents were forcing her to go to school and threatening to run away. She also locked her younger siblings out of the house in cold weather. Although her parents attempt to set limits, she frequently responds with prolonged tantrums- crying and shouting, flailing her arms, and breaking objects in her home. Once she "gets her way" she settles down immediatley. After one serious tantrum, her parents took her to the emergency department and the girl was admitted to the psychiatric unit. The pt was viewed as quite charming by several members of the staff, whereas others though she was "a very, very difficult pt." What is the likely diagnosis? A. Attention- deficit/ hyperactivity disorder. B. Conduct disorder. C. Intermittent explosive disorder. D. Oppositional defiant disorder.

D. Oppositional defiant disorder. This disorder is represented by angry/ irritable mood, argumentative/ defiant behavior's and vindictiveness that is not typical for the child's developmental level. In children younger than age 5, the behavior's must be present on most days for at least 6 months. In children older than 5 it must occur at least once per week for at least 6 months.

A 12- year- old girl is getting along poorly with her parents and teachers. She is highly argumentative and defiant at home. She has attempted to embarrass her teacher in front of the other students and uses profane language. She never seems to take responsibility for the hostility that appears to be present in many of her social interactions. Due to her constant "negativity" with her teachers, other students have begun to keep their distance from her. In the past 5 years, the girl's parents have gotten divorced. The girl is cooperative in responding to questions during the psychiatric interview. What is the likely diagnosis? A. Antisocial personality disorder . B. Conduct disorder. C. Major depressive disorder. D. Oppositional defiant disorder.

D. Oppositional defiant disorder. This disorder is represented by angry/ irritable mood, argumentative/ defiant behavior's and vindictiveness that is not typical for the child's developmental level. In children younger than age 5, the behaviors must be press on most days for at least 6 months. In children older than 5 it must occur at least once per week for at least 6 months.

A 62- year- old veteran underwent lung transplantation for idiopathic pulmonary fibrosis 6 months ago. His postoperative course has been complicated by a prolonged stay in the intensive care unit, renal insufficiency dependent on dialysis, atrial fibrillation, pneumonia, and severe deconditioning. He has difficulty working on his strengthening with physical therapy b/c he becomes anxious and worries about his perceived shortness of breath. He frequently asks to avoid or postpone these sessions and has been described as noncooperative with his treatment. This behavior, in turn, impedes his recovery. On evaluation by psychiatry, the pt denies depressed mood, reports enjoying visits from his family and watching the news, and is hopeful about his recovery. What is the likely diagnosis? A. Adjustment disorder. B. Generalized anxiety disorder. C. Posttraumatic stress disorder. D. Psychological factors affecting other medical condition.

D. Psychological factors affecting other medical condition.

Which developmental stage represents the usual time of onset of pedophilic disorder in males? A. Childhood. B. Late life. C. Middle life. D. Puberty.

D. Puberty.

Police arrest a 21- year- old man for arson, and a psychiatrist evaluates him in jail. On interview, the man describes having had an intense fascination with fire since his teenage years. He feels excited whenever he lights matches and enjoys watching the flames "dance" before him. He denies a hx of unstable relationships, getting into fights, or deceiving others. He denies any suicidal or homicidal ideation, attempts, or gestures. His record shows not other legal charges besides arson. He denies hallucinations or unstable mood, and his thought process is linear. What is the likely diagnosis? A. Borderline personality disorder. B. Conduct disorder. C. Delusional disorder. D. Pyromania.

D. Pyromania. This disorder is the deliberate setting of fires on more than one occasion and the experiencing of pleasure or relief while setting the fires or in the aftermath of the fire setting. Behavior involves a fascination with or attraction to fire. The individual spends significant time planning, setting and watching fires. The person fails to resist an impulse to set fires. Individuals may be indifferent to the consequences of the fire and its effect on life and property.

The emergency department calls a psychiatrist to evaluate a 35- year- old homeless, English- speaking man, identifying him as Native American. The police brought in the man after bizarre posturing in a park while talking to himself. The man describes having special relationships with animals. He believes that he can communicate with them through telepathy. He reports that this communication with nature is a common belief in his culture and that he has this special "gift". He says that he receives "special signals" from birds that influence his behavior and he can "hear them talk to him". Under further exploration, he reports that he has mental powers that influence animals and nature. He insists that he is "not crazy". His hygiene is poor and he is malodorous. He has very odd habits around eating based on his auditory hallucinations, resulting in poor nutrition and weight loss. He looks emaciated. He does not have a reasonable plan for shelter and sleeps in a field. His clothes are smiled. He has odd mannerisms throughout the interview, including laughing inappropriately. His urine drug toxicology is negative for substances of abuse. He has a hx of multiple hospitalizations and involuntary commitments for grave disability starting around age 22 years. What is the likely diagnosis? A. Alcohol use disorder. B. Culturally appropriate behavior. C. Major depressive disorder. D. Schizoprehinia.

D. Schizophrenia

Theravada Buddhist monks living at a temple bring a 27- year-old Laotian man to the hospital. The man apparently has fasted for 17 days. The man is acutely dehydrated and emaciated. The pt is admitted to the medicine service. He is delirious at first, but when he improves, the psychiatric consult service is called b/c of his "odd manner". The pt is unkempt and unshaven, which is in sharp contrast to the two other members of his order. The pt does not speak English and one of the monks translates to help the psychiatrist with the evaluation. The pt appears withdrawn and responds very briefly to the other monks, at times laughing inappropriately. The monks say that the man's family, who had recently emigrated from Laos, brought him to the order nearly 1 year ago. Although he has been withdrawn since they have known him, the monks say that this is the first time he has embarked on such a fast. He told them that he was instructed to do so by "spirits", who watch his daily activities and comment on whether he is being appropriately observant. The members of his order are familiar with the names of the spirits from Lao folk traditions, but they are not familiar with this particular tradition of fasting. They acknowledge that the man is from a remote part of Laos, of which the are unfamiliar with the customs. What is the likely diagnosis? A. Culturally appropriate behavior. B. Major depressive disorder. C. Psychotic disorder due to another medical condition. D. Schizophrenia.

D. Schizophrenia. Is a psychotic disorder characterized by a combination of specific positive and negative symptoms. Positive symptoms of schizophrenia include hallucinations, delusions, disorganized speech, inappropriate affect, and disorganized behavior (absence of goal- orientation often manifested in failure to perform activities of daily living). Negative symptoms of schizophrenia include a flat or blunted affect (the absence or a severe restriction of observable emotion), avolition (a loss of willpower and decisiveness), alogia (a speech disturbance in schizophrenia involving poverty of speech), decrease in the amount of speech or poverty of content of speech (speech that contains little or no meaningful information), and anhedonia (a loss of ability to experience pleasure).

Match each description with the personality disorder for which it is most highly characteristic (each disorder may be used once, more than once, or not at all): A. Avoidant personality disorder. B. Paranoid personality disorder. C. Schizoid personality disorder. D. Schizotypal personality disorder. _ Behavior that is odd, eccentric, or peculiar, with odd beliefs or magical thinking that influences behavior and are inconsistent with cultural norms. _ Neither desires nor enjoys close relationships, with emotional coldness, detachment, or flattened affective responses to others. _ Suspects that others are exploiting or deceiving him or her, and persistently bears a grudge against others for perceived insults or slights.

D. Schizotypal personality disorder- Behavior that is odd, eccentric, or percolating, with odd beliefs or magical thinking that influence behavior and are inconsistent with cultural norms. C. Schizoid personality disorder- Neither desires nor enjoys close relationships, with emotional coldness, detachment, or flattened affective responses to others. B. Paranoid personality disorder- Suspects that others are exploiting or deceiving him or her, and persistently bears a grudge against others for perceived insults or slights.

A 19- year- old college freshman wears a wizard hat every day around campus. When asked why, he tells people it is because "the hat helps me think better." He has had few friends but enjoys participating in medieval role- playing games. He earns excellent grades in class and grooms himself appropriately every day. His speech is fluent. His thought process is linear, although he perseverates on various wizard spells he has learned online. His affect is restricted. He denies any hallucinations or any changes in sleep, mood, appetitie, or energy. His unusual "style" has interfered with his getting a job, which he needs to remain in school. What is the likely diagnosis? A. Bipolar I disorder. B. Schizoaffective disorder. C. Schizophrenia D. Schizotypal personality disorder.

D. Schizotypal personality disorder. This disorder is characterized by a pattern of deficits in interpersonal skills and a decreased capacity for close relationships. Cognitive and perceptual distortions and eccentric behavior are also evident.

A mother brings her 15- year- old daughter to a psychiatrist's office. The mother says that for the past year, her daughter has not said a word to her teachers. With her family members or friends, however, she is very talkative and engaging, without any speech impairments or language abnormalities. The mother notes that her daughter met all developmental milestones appropriately and reached puberty at age 14. She never had to repeat a grade in school, although her grades this year are faltering b/c she does not speak in class. She is otherwise attentive and does well on homework. The mother denies any impulsive behavior. The pt does not speak to the psychiatrist when addressed, but shakes her head no when asked if she has experienced any hallucinations, mood disturbances, or traumatic episodes in her past. She sits patiently in her chair, smiling, without any fidgeting or unusual movements. What is the likely diagnosis? A. Attention- deficit/ hyperactivity disorder. B. Autism spectrum disorder. C. Schizophrenia D. Selective mutism.

D. Selective mutism. This disorder is identified by a persistent failure to speak in certain situations with a demonstrated ability to talk as evidenced by doing so in other situations. Selective mutism last a minimum of 1 month and cannot be evident only in the first month of school. This diagnosis is not given if it is apparent that the individual's failure to speak has its roots in a lack of familiarity of or comfort with the language. Selective mutism is slightly more common in females.

A 40- year- old woman says she is "very anxious about public speaking", to the point of having episodes that resemble panic attacks when she faces speaking before even small groups of familiar coworkers. Her division chief expects that employees will present at conferences before he will promote them. She is not only unable to give presentations to large audiences but also avoids most social events related to her work. She feels as if she is "acting like an idiot" when she is with colleagues, especially her work supervisors. The anticipation of a meeting with her boss can cause her to worry and lose sleep for days. She does not have anxiety in other contexts. What is the likely diagnosis? A. Generalized anxiety disorder. B. Major depressive disorder. C. Obsessive- compulsive personality disorder. D. Social anxiety disorder.

D. Social anxiety disorder. This disorder is characterized by far of embarrassing oneself in social situations or feeling foolish. Avoidance behavior is often evident and the condition is not due to a medical situation or substance. The disorder typically lasts for more than 6 months.

A 32- year- old woman presents to the emergency department, requesting help for back pain. She says that she has wrestled with this pain for 2 years, after a minor skiing accident, and no medications or interventions have helped. No physical evidence explains the degree of pain she reports. The pain prevents her from working as a ballerina. She loves her job but says that for the past few years, budget cuts have stressed the entire ballet company. She denies any problems with sleep, anxiety, or mood. Her body weight is 90% of expected, and she denies any problems with her weight or diet. She is cooperative, and her thought process is linear. What is the likely diagnosis? A. Adjustment disorder. B. Anorexia nervosa. C. Major depressive disorder. D. Somatic symptom disorder, with predominant pain.

D. Somatic symptom disorder, with predominant pain This disorder is characterized by six or more months of a general preoccupation with fears of having a serious disease that has not been detected based on the individual's misinterpretation of bodily symptoms (previously known as hypchondriasis. The conviction persists despite negative physical and laboratory findings. It causes clinically significant distress and impairment.

A mother brings her 6- year- old son to the pediatrician b/c of difficulty at school. His mathematical abilities are below his grade level, but he is otherwise an average student in all his other classes. His mother denies any problems with hyperactivity and impulsivity, and he is social with peers at school. He does not have behavioral problems at school, and his teachers say he is a pleasant and attentive student. He met all his developmental milestones appropriately. What is the likely diagnosis? A. Attention- deficit/ hyperactivity disorder. B. Autism spectrum disorder. C. Global developmental delay. D. Specific learning disorder.

D. Specific learning disorder. This disorder involves a persistent inability to learn or use academic skills as quickly or as accurately as other children of the same developmental age. The specific learning difficulties occur during school- age years and my not manifest until the demands of school exceed the individual's abilities. The disorder significantly impairs the person's ability to perform activities of daily living or in academic areas. This disorder tens to be more common in males.

A 41- year- old man reports to the psychiatrist that he is afraid of dentists. He has a hx of extreme anxiety upon entering a dental office, to the point that he feels dizzy and nauseated. He becomes excessively anxious in the waiting room and has a very difficult time completing visits. He is so fearful about dentists that he reports he cannot take his own children to their dentist b/c he has "anxiety attacks" in the office. Rather, he insists that his wife take the children. His aversion to the dentists has led to his missing regular appointments for more than 6 years. He plans to go soon, only b/c he has severe tooth pain and believes he may need a root canal. He does not have anxiety in other contexts. What is the likely diagnosis? A. Adjustment disorder with anxiety. B. Generalized anxiety disorder. C. Posttraumatic stress disorder. D. Specific phobia.

D. Specific phobia.

A man brings his 36- year- old wife to a new primary care doctor b/c she has lost patches of her hair recently. He notes that her last doctor sent her to a dermatologist, who felt that an underlying skin condition was unlikely. In a private exam room without her husband present, she says that she secretly pulls out her hair in clumps to relieve tension. The act of pulling her hair brings her relief. She denies changes in mood, sleep, concentration,, or energy level. What is the likely diagnosis? A. Generalized anxiety disorder. B. Major depressive disorder. C. Tic disorder. D. Trichotillomania

D. Trichotillomania (Hair - pulling disorder) This disorder involves compulsive hair pulling that leads to significant hair loss. There is increased tension before hair pulling and pleasure or relief when pulling the hair out. The individual makes repeated attempts to decrease or stop the hair pulling. This behavior is not the result of a medical condition or another mental condition. This condition is more common in women and the average age of onset is 13 year of age.


Kaugnay na mga set ng pag-aaral

Unit 4 English Which sentence uses numbers correctly?

View Set

Chapter 11 - Stockholders' Equity

View Set

Fundamentals Criminal Law quiz 3

View Set

practice for practice google ads test

View Set

GCSE Evidence for Climate Change

View Set